Dirac delta function definition

Click For Summary
The Dirac delta function is defined such that the integral of a function multiplied by the delta function evaluates to the function's value at a specific point. In the context of the discussion, the expression involving the curl of the magnetic field and the current density leads to an integral that incorporates the delta function. The confusion arises regarding the argument of the delta function, specifically whether it should be δ(r' - r) or δ(r - r'). It is clarified that both forms are equivalent due to the symmetry of the delta function, as it spikes at the same point where the arguments are equal. Thus, the integration ultimately yields the desired result of μ₀J(r).
latentcorpse
Messages
1,411
Reaction score
0
By definition of the Dirac delta function, we have:

\int f(x) \delta(x-a) dx=f(a)

This is fair enough. But in ym notes there is a step that goes like the following:

\mathbf{\nabla} \wedge \mathbf{B}(\mathbf{r})=-\frac{\mu_0}{4 \pi} \int_V dV' \nabla^2(\frac{1}{|\mathbf{r}-\mathbf{r'}|}) \mathbf{J}(\mathbf{r'}) = \mu_0 \mathbf{J}(\mathbf{r})

where we have used that \nabla^2(\frac{1}{|\mathbf{r}-\mathbf{r'}|}) =-4 \pi \delta(\mathbf{r}-\mathbf{r'})

clearly the minus signs and the 4 \pi's cancel so it's now just

\mathbf{\nabla} \wedge \mathbf{B}(\mathbf{r})=\mu_0 \int_V dV' \mathbf{J}(\mathbf{r'}) \delta(\mathbf{r}-\mathbf{r'})

i don't see how that goes from there to give \mu_0 \mathbf{J}(\mathbf{r}) as we are integrating with respect to V' are we not? and so i would assume that the delta function would need to be of the form \delta(\mathbf{r'}-\mathbf{r}) in order to give the desired answer.

the onnly explanation i can come up with is that \delta(\mathbf{r'}-\mathbf{r})=\delta(\mathbf{r}-\mathbf{r'}) since the delta function is symmetric about the spike. however the spike would be at different positions in these two cases. I'm kind of lost-any advice?
 
Physics news on Phys.org
latentcorpse said:
the onnly explanation i can come up with is that \delta(\mathbf{r'}-\mathbf{r})=\delta(\mathbf{r}-\mathbf{r'}) since the delta function is symmetric about the spike.

Yes, that's true.

however the spike would be at different positions in these two cases. I'm kind of lost-any advice?

No, the delta function has a spike whenever it's argument is zero. In both cases this occurs at r=r'.
 
Question: A clock's minute hand has length 4 and its hour hand has length 3. What is the distance between the tips at the moment when it is increasing most rapidly?(Putnam Exam Question) Answer: Making assumption that both the hands moves at constant angular velocities, the answer is ## \sqrt{7} .## But don't you think this assumption is somewhat doubtful and wrong?

Similar threads

  • · Replies 5 ·
Replies
5
Views
745
  • · Replies 3 ·
Replies
3
Views
616
  • · Replies 1 ·
Replies
1
Views
533
  • · Replies 2 ·
Replies
2
Views
3K
  • · Replies 29 ·
Replies
29
Views
1K
  • · Replies 7 ·
Replies
7
Views
2K
  • · Replies 2 ·
Replies
2
Views
2K
  • · Replies 1 ·
Replies
1
Views
1K
  • · Replies 2 ·
Replies
2
Views
1K
  • · Replies 2 ·
Replies
2
Views
903